Quiz 7 Solution


1. Which of these is (are) improper?
\begin{array}{ll}
a. $\int_0^\infty e^{-x}\,dx$\space & b. $\int_{-1}^{1}\frac{d...
...1}\sqrt{1-x^2}\,dx$\space & d. $\int_{-1}^{1}\frac{dx}{\sqrt{1-x^2}}$\end{array}


See the definition of an improper integral, Thomas/Finney p. 595. 'a' is clearly improper. So is 'b' since the integrand goes to infinity between -1 and 1 (That is, as x approaches 0, the integrand goes to infinity). Again, so is 'd' since it's integrand goes to infinity at the endpoints.


2.

\begin{displaymath}I=\int_0^3 x^r\,dx = \left\{ \begin{array}{ll}
\left. \frac{x...
... \ln x \right\vert _0^3 &\mbox{if $r=-1$ }
\end{array} \right. \end{displaymath}

case 1: $r\ne -1$
Then

\begin{displaymath}I = \frac{3^{r+1}}{r+1} - \frac{0^{r+1}}{r+1} \end{displaymath}

The first term is easy to calculate since $r\ne -1$. If r>-1, then r+1>0, and so the second term is zero. But if r<-1, we have r+1<0 and so the second term is undefined (but goes to infinity if we take the limit). So, in this case, the integral converges if and only if r>-1.


case 2: r=-1
Then

\begin{displaymath}I = \ln 3 - \ln 0 \end{displaymath}

Again, the first term is ok. But the second term is undefined, and if we take the limit, it goes to $-\infty$. Therefore, the integral does not converge in this case.


So, by these 2 cases, we see that the integral converges if and only if r>-1.


 

Christopher John Monico
1999-03-17
You are visitor